A very hard, above 700 DS question from OG 12

This topic has expert replies
Senior | Next Rank: 100 Posts
Posts: 95
Joined: Thu Mar 12, 2009 2:39 am
GMAT Score:620
Below is the question from OG, a real tough one. I am totally clueless on this one. Please help me out. Thanks in advance

Beginning in January of last year, Carl made deposits
of $120 into his account on the 15th of each month
for several consecutive months and then made
withdrawals of $50 from the account on the 15th of
each of the remaining months of last year. There were
no other transactions in the account last year. If the
closing balance of Carl's account for May of last year
was $2,600, what was the range of the monthly
closing balances of Carl's account last year?

(1) Last year the closing balance of Carl's account
for April was less than $2,625.

(2) Last year the closing balance of Carl's account
for June was less than $2,675

Master | Next Rank: 500 Posts
Posts: 399
Joined: Wed Apr 15, 2009 3:48 am
Location: india
Thanked: 39 times

by xcusemeplz2009 » Mon Oct 26, 2009 9:11 am
is it E?
It does not matter how many times you get knocked down , but how many times you get up

Senior | Next Rank: 100 Posts
Posts: 50
Joined: Tue Oct 13, 2009 1:25 am
Thanked: 14 times
GMAT Score:720

by November Rain » Mon Oct 26, 2009 10:08 am
I would say B, but i am not sure about the logic that i followed.

What is the OA?

Junior | Next Rank: 30 Posts
Posts: 15
Joined: Mon Aug 03, 2009 2:19 pm

by deepak115 » Mon Oct 26, 2009 11:10 am
Either Stmt D

Senior | Next Rank: 100 Posts
Posts: 41
Joined: Tue Oct 06, 2009 9:49 am
Thanked: 4 times
gmat620 wrote:Below is the question from OG, a real tough one. I am totally clueless on this one. Please help me out. Thanks in advance

Beginning in January of last year, Carl made deposits
of $120 into his account on the 15th of each month
for several consecutive months and then made
withdrawals of $50 from the account on the 15th of
each of the remaining months of last year. There were
no other transactions in the account last year. If the
closing balance of Carl's account for May of last year
was $2,600, what was the range of the monthly
closing balances of Carl's account last year?

(1) Last year the closing balance of Carl's account
for April was less than $2,625.

(2) Last year the closing balance of Carl's account
for June was less than $2,675
I think its D.

To determine the range here we wil need to know exactly when the deposits stopped and withdrawls started. We already know May balance so we can find out balance of each month IF we know when the withdrawls stopped.

If in May there was a deposit, then april balance would be 2600 - 120 = 2480 and if there was a withdrawl then the balance would have been 2600 + 50 = 2650

If we take statement 1 then 2650 is ruled out. So there WAS a DEPOSIT in May.

Now for June if there was a deposit, the balance would be 2600 + 120 = 2720
If there was a withdrawl then June balance would be 2600 - 50 = 2550

If we take statement-2 then 2720 is ruled out so there was a WITHDRAWL in June.

Putting both statements together, we know that there was a deposit in May and withdrawl in June.

This information is enough to know all the remaining month balances. THerefore we can calculate the range.

Senior | Next Rank: 100 Posts
Posts: 95
Joined: Thu Mar 12, 2009 2:39 am
GMAT Score:620

by gmat620 » Tue Oct 27, 2009 2:58 am
Thanks a lot friends, OA is C

User avatar
Senior | Next Rank: 100 Posts
Posts: 44
Joined: Sun Dec 27, 2009 8:50 am
Thanked: 3 times
GMAT Score:710

by skprocks » Fri Aug 20, 2010 10:54 pm
gmat620 wrote:Below is the question from OG, a real tough one. I am totally clueless on this one. Please help me out. Thanks in advance

Beginning in January of last year, Carl made deposits
of $120 into his account on the 15th of each month
for several consecutive months and then made
withdrawals of $50 from the account on the 15th of
each of the remaining months of last year. There were
no other transactions in the account last year. If the
closing balance of Carl's account for May of last year
was $2,600, what was the range of the monthly
closing balances of Carl's account last year?

(1) Last year the closing balance of Carl's account
for April was less than $2,625.

(2) Last year the closing balance of Carl's account
for June was less than $2,675
This thread does not seem to clarify the problem at hand.I will try to post an explannation that helps.First,things first.The most relevant part of this question is that
once withdrawal starts it continues for the rest of the year.And Deposits started prior to the withdrawals.Now,Look at the Statements.

Stmt1: Balance in April is < 2625. Its already known that the balance in May was 2600.5 consecutive deposits of 120 make just 600.
So, Brought Forward/Initial amount in Jan prior to any deposits was 2000.Since May was 2600,so April was also a deposit month.
As had may been withdrawal the amount in april shall be 2650 for May to be 2600,as withdrawals are in 50 denominations.
Thus April was a month when deposit was made.But this stmt gives no clue about when withdrawals began. INSUFFICIENT

Stm2: June balance was < 2675.Had june been a deposit month its balance wud be 2600+120=2720.But this stmt indicates otherwise,
so withdrawals began in June and continued till December.Now the fact indicated by this statement is just that withdrwals began in June.
Does not give sufficient info to assume Jan initial balance as 2000 et al.INSUFFICIENT

Combining Stmt1 and Stmt2 we get that Deposits continued till May and Withdrawals began in June.June onwards balance continuously falls
by50 each month.This gives the balance at the end of each month,Hence its possible to determine Range= Max- Min balance.
Jan->2120,2240,2360,2480,2600,2550,2500,2450,2400,2350,2300,2250.
Range = 2600-2120=480

Junior | Next Rank: 30 Posts
Posts: 25
Joined: Sat Nov 12, 2011 6:13 am
Followed by:1 members

by Cumulonimbus » Wed Apr 24, 2013 9:24 pm
I solved this one correctly...but took more time...dont know what strategy to use on exam day

User avatar
GMAT Instructor
Posts: 15539
Joined: Tue May 25, 2010 12:04 pm
Location: New York, NY
Thanked: 13060 times
Followed by:1906 members
GMAT Score:790

by GMATGuruNY » Thu Apr 25, 2013 2:41 am
Beginning in January of last year, Carl made deposits
of $120 into his account on the 15th of each month
for several consecutive months and then made
withdrawals of $50 from the account on the 15th of
each of the remaining months of last year. There were
no other transactions in the account last year. If the
closing balance of Carl's account for May of last year
was $2,600, what was the range of the monthly
closing balances of Carl's account last year?

(1) Last year the closing balance of Carl's account for April was less than $2,625.
(2) Last year the closing balance of Carl's account for June was less than $2,675.
May = 2600.

Statement 1: Last year the closing balance of Carl's account
for April was less than $2,625.

If 120 was deposited in May, then April = 2600-120 = 2480.
If 50 was withdrawn in May, then April = 2600+50 = 2650.
Since April < 2625, we know that a deposit of 120 was made in May, implying that 120 was deposited every month January through at least May.
Thus, we can determine the balance of every month January - April:
April = 2480, March = 2480-120 = 2360, etc.
No information about June - December.
Insufficient.

Statement 2: Last year the closing balance of Carl's account
for June was less than $2,675.

If 120 was deposited in June, then June = 2600+120 = 2720.
If 50 was withdrawn in June, then June = 2600-50 = 2550.
Since June < 2675, we know that a withdrawal of 50 was made in June, implying that 50 was withdrawn every month June - December.
Thus, we can determine the balance of every month June - December:
June = 2550, July = 2550-50 = 2500, etc.
No information about January - April.
Insufficient.

Statements 1 and 2 combined:
Statement 1 tells us the closing balances January - April.
The question stem tells us the closing balance in May.
Statement 2 tells us the closing balances June - December.
Combining the information above, we can determine the closing balance of every month and determine the range for the whole year.
Sufficient.

The correct answer is C.
Private tutor exclusively for the GMAT and GRE, with over 20 years of experience.
Followed here and elsewhere by over 1900 test-takers.
I have worked with students based in the US, Australia, Taiwan, China, Tajikistan, Kuwait, Saudi Arabia -- a long list of countries.
My students have been admitted to HBS, CBS, Tuck, Yale, Stern, Fuqua -- a long list of top programs.

As a tutor, I don't simply teach you how I would approach problems.
I unlock the best way for YOU to solve problems.

For more information, please email me (Mitch Hunt) at [email protected].
Student Review #1
Student Review #2
Student Review #3

User avatar
Legendary Member
Posts: 1100
Joined: Sat May 10, 2014 11:34 pm
Location: New Delhi, India
Thanked: 205 times
Followed by:24 members

by GMATinsight » Mon Oct 12, 2015 3:24 am
Beginning in January of last year, Carl made deposits of $120 into his account on the 15th of each month for several consecutive months and then made withdrawals of $50 from the account on the 15th of each of the remaining months of last year. There were no other transactions in the account last year. if the closing balance of Carl's account for May of last year was $2,600, what was the range of the monthly closing balances of Carl's account last year?

(1) Last year the closing balance of Carl's account for April was less than $2,625.

(2) Last year the closing balance of Carl's account for June was less than $2,675.
Given :
Jan
Feb
Mar
Apr
May - $2600
Jun
Jul
Aug
Sep
Oct
Nov
Dec

Statement 1: April < 2625
Since either $50 withdrawl or $120 deposit is possible so for withdraw of $50 in May, April must have been $2600+50 = 2650 for a deposit of $50 which is not true as per statement 1 i.e. Deposit of $120 must have happened in all the months before MAY till MAY
Jan - $2120
Feb - $2240
Mar - $2360
Apr - $2480

May - $2600
Jun
Jul
Aug
Sep
Oct
Nov
Dec
It gives us the amount from Jan to may but not for months later so
NOT SUFFICIENT

Statement 2: June < 2675
Since either $50 withdrawl or $120 deposit is possible so for withdraw of $50 in May, June must have been $2600+120 = $2720 for a deposit of $120 which is not true as per statement 2 i.e. Withdrawl of $50 must have happened in all the months After MAY
Jan
Feb
Mar
Apr
May - $2600
Jun - $2550
Jul - $2500
Aug - $2450
Sep - $2400
Oct - $2350
Nov - $2300
Dec - $2250


It gives us the amount from May to Dec but not for months before MAY so
NOT SUFFICIENT

Combining the two statements

Jan - $2120
Feb - $2240
Mar - $2360
Apr - $2480

May - $2600
Jun - $2550
Jul - $2500
Aug - $2450
Sep - $2400
Oct - $2350
Nov - $2300
Dec - $2250

SUFFICIENT

Answer: option C
"GMATinsight"Bhoopendra Singh & Sushma Jha
Most Comprehensive and Affordable Video Course 2000+ CONCEPT Videos and Video Solutions
Whatsapp/Mobile: +91-9999687183 l [email protected]
Contact for One-on-One FREE ONLINE DEMO Class Call/e-mail
Most Efficient and affordable One-On-One Private tutoring fee - US$40-50 per hour

GMAT/MBA Expert

User avatar
GMAT Instructor
Posts: 16207
Joined: Mon Dec 08, 2008 6:26 pm
Location: Vancouver, BC
Thanked: 5254 times
Followed by:1268 members
GMAT Score:770

by Brent@GMATPrepNow » Mon Oct 12, 2015 9:24 am
gmat620 wrote:Below is the question from OG, a real tough one. I am totally clueless on this one. Please help me out. Thanks in advance

Beginning in January of last year, Carl made deposits
of $120 into his account on the 15th of each month
for several consecutive months and then made
withdrawals of $50 from the account on the 15th of
each of the remaining months of last year. There were
no other transactions in the account last year. If the
closing balance of Carl's account for May of last year
was $2,600, what was the range of the monthly
closing balances of Carl's account last year?

(1) Last year the closing balance of Carl's account
for April was less than $2,625.

(2) Last year the closing balance of Carl's account
for June was less than $2,675
Target question: What was the range of the monthly closing balances of Carl's account last year?

Given: The closing balance of Carl's account for May of last year was $2,600

IMPORTANT: To answer the target question we need only determine which month Carl STARTED withdrawing money.
For example, if he started withdrawing money on March 15, we could use the fact that he had $2600 at the end of May to determine how much he had in the bank every month of the year, and thus determine the range of closing balances.
Notice that, since this is a Data Sufficiency, we need not calculate the actual range. We need only determine which month the deposits stopped and the withdrawals started.

So, we can rephrase our target question as . . .

REPHRASED target question: In which month did Carl start withdrawing $50?

Statement 1: Last year the closing balance of Carl's account for April was less than $2,625
Let's examine two cases:
case a: In May, Carl DEPOSITED $120. So, balance at end of April = $2600 - $120 = $2480. This is possible, since we're told that the balance is less than $2625
case b: In May, Carl WITHDREW $50. So, balance at end of April = $2600 + $50 = $2650. This is NOT possible, since we're told that the balance is less than $2625
So, Carl definitely deposited $120 in May (and deposited $120 in April, March, Feb, and Jan).
However, we don't know the first month that Carl started withdrawing $50
Since we cannot answer the REPHRASED target question with certainty, statement 1 is NOT SUFFICIENT

Statement 2: Last year the closing balance of Carl's account for June was less than $2,675.
Let's examine two cases:
case a: In June, Carl DEPOSITED $120. So, balance at end of June = $2600 + $120 = $2720. This is NOT possible, since we're told that the balance is less than $2675
case b: In June, Carl WITHDREW $50. So, balance at end of June = $2600 - $50 = $2550. This is possible, since we're told that the balance is less than $2675
So, Carl definitely withdrew $50 in June, which means he also withdrew $50 in July, August, Sept, etc. However, we don't know the FIRST month that Carl started withdrawing $50
Since we cannot answer the REPHRASED target question with certainty, statement 2 is NOT SUFFICIENT

Statements 1 and 2 combined
Statement 1 tells us that Carl deposited $120 in May.
Statement 2 tells us that Carl withdrew $50 in June.
So, June was the first month that Carl started withdrawing $50
Since we can answer the REPHRASED target question with certainty, the combined statements are SUFFICIENT

Answer = C

Cheers,
Brent
Brent Hanneson - Creator of GMATPrepNow.com
Image

Master | Next Rank: 500 Posts
Posts: 137
Joined: Fri Nov 13, 2015 11:01 am
Thanked: 1 times
Followed by:2 members

by Amrabdelnaby » Sat Jan 02, 2016 2:32 pm
I think the logic behind this question is very simple although it appears from the first glance to be terribly difficult.

basically statement 1 tells us what happened from jan to may (deposits) but doesn't tell us what happened afterwards.

and statement two tells us what happened from june untill the end of the year.

hence statement one and two combined give us an insight about the withdrawals and deposits hence we know how much he had left in the account by the end of the year and how much he had in the beginning and hence we can find the range.

I have noticed that all data sufficiency questions use this logic, one statement pushes the answer choice in one direction; sometimes it's enough and sometimes it needs another statement to corner the answer choice in one place. Or both answer choices fail to yield one possible result
GMATGuruNY wrote:
Beginning in January of last year, Carl made deposits
of $120 into his account on the 15th of each month
for several consecutive months and then made
withdrawals of $50 from the account on the 15th of
each of the remaining months of last year. There were
no other transactions in the account last year. If the
closing balance of Carl's account for May of last year
was $2,600, what was the range of the monthly
closing balances of Carl's account last year?

(1) Last year the closing balance of Carl's account for April was less than $2,625.
(2) Last year the closing balance of Carl's account for June was less than $2,675.
May = 2600.

Statement 1: Last year the closing balance of Carl's account
for April was less than $2,625.

If 120 was deposited in May, then April = 2600-120 = 2480.
If 50 was withdrawn in May, then April = 2600+50 = 2650.
Since April < 2625, we know that a deposit of 120 was made in May, implying that 120 was deposited every month January through at least May.
Thus, we can determine the balance of every month January - April:
April = 2480, March = 2480-120 = 2360, etc.
No information about June - December.
Insufficient.

Statement 2: Last year the closing balance of Carl's account
for June was less than $2,675.

If 120 was deposited in June, then June = 2600+120 = 2720.
If 50 was withdrawn in June, then June = 2600-50 = 2550.
Since June < 2675, we know that a withdrawal of 50 was made in June, implying that 50 was withdrawn every month June - December.
Thus, we can determine the balance of every month June - December:
June = 2550, July = 2550-50 = 2500, etc.
No information about January - April.
Insufficient.

Statements 1 and 2 combined:
Statement 1 tells us the closing balances January - April.
The question stem tells us the closing balance in May.
Statement 2 tells us the closing balances June - December.
Combining the information above, we can determine the closing balance of every month and determine the range for the whole year.
Sufficient.

The correct answer is C.

GMAT Instructor
Posts: 2630
Joined: Wed Sep 12, 2012 3:32 pm
Location: East Bay all the way
Thanked: 625 times
Followed by:119 members
GMAT Score:780

by Matt@VeritasPrep » Fri Jan 08, 2016 2:07 pm
Amrabdelnaby wrote:I think the logic behind this question is very simple although it appears from the first glance to be terribly difficult.
The wording is pretty boring and dry: I think the idea is to test your ability to focus on very literal descriptions of financial transactions, something that many/most MBAs will have to deal with during their classes and afterwards. But it isn't all that difficult or convoluted if you're able to stay awake while reading it (easier said than done! :D)